Đến nội dung

Hình ảnh

$\frac{(x_1+2x_2+...+nx_n)(x_1+\frac{x_2}{2}+...+\frac{x_n}{n})}{(x_1+x_2+...+x_n)^2} \leq \frac{(n+1)^2}{4n}$

- - - - -

  • Please log in to reply
Chưa có bài trả lời

#1
MiuraHaruma

MiuraHaruma

    Binh nhất

  • Thành viên
  • 25 Bài viết
(Chọn Đội tuyển Quảng Bình 2014) Cho $x_1, x_2, ..., x_n \geq 0$. Cmr:
$\frac{(x_1+2x_2+...+nx_n)(x_1+\frac{x_2}{2}+...+\frac{x_n}{n})}{(x_1+x_2+...+x_n)^2} \leq \frac{(n+1)^2}{4n}$
"Every saint has a past, every sinner has a future"




1 người đang xem chủ đề

0 thành viên, 1 khách, 0 thành viên ẩn danh